MBE Simulated Exam Flashcards

1
Q

Federal narcotics officers suspected the defendant of growing marijuana in his green- house, which was connected to his house. The narcotics officers learned from an anonymous informant that the semi-opaque panes of glass on the greenhouse were being replaced during the night with a newer type of glass that let in more light without an increase in visibility. Without
a warrant, the officers flew over the defendant’s greenhouse in a helicopter that night. One of the officers focused on the greenhouse with a pair
of “night vision” thermal imaging binoculars supplied by the Department of Defense and not available to the general public. He determined that marijuana was being grown. The officers then went to a magistrate, swore out a warrant, and arrested the defendant.

If the defendant moves to suppress any evidence gathered by virtue of the flyover, how should the court rule on the motion?

(A) Deny it, because the police may conduct flyovers to gather evidence.

(B) Deny it, because the defendant did not live in the greenhouse.

(C) Grant it, because the “night-vision” binocu- lars were not available to the general public.

(D) Grant it, because the flyover was prompted by a tip from an anonymous informant.

A

(C) The use of thermal imaging binoculars to observe the marijuana where it could not be observed by simply using the naked eye likely renders the search invalid. To be able to assert a Fourth Amendment right, a person must have a reasonable expectation of privacy with respect to the place searched or the item seized. There is no such expectation of privacy in objects or places held out to the public or that may be viewed from a public vantage point. Thus, the police may fly over an area to observe it with the naked eye, and even a low flyover by a helicopter to view inside a partially covered building is permissible. This is true even if the area is within the curtilage. However, the police may not use technologically enhanced methods that are not available to the public to search areas (at least as to areas within the curtilage). In the instant case, the police have flown over the defendant’s greenhouse at night and used a means of enhancing their vision that is not available to the general public. This enabled them to see what could not have been observed with the naked eye, which likely constitutes an impermissible search.

(A) is incorrect because it is too broad a statement. The police generally may look into any area, even an area within the home or curtilage, by means of a flyover, so long as they do so from a place the public has access to; however, the police may not use technological enhance- ments that are not available to the public. In the instant case, the use of thermal imaging makes the police conduct unconstitutional.

How well did you know this?
1
Not at all
2
3
4
5
Perfectly
2
Q

A large insurance company instituted a supplemental benefit plan for its own employees. Under the plan, any employee who had worked for the company for at least 25 years would be permitted to designate a charity to receive, on the employee’s retirement, a donation in the employee’s name of six months’ worth of the employee’s salary. The plan gave participating employees an unqualified right to change the beneficiary at any time before payment was made. An employee nearing retirement enrolled in the plan and named his favorite church as the beneficiary of the donation. The church received a letter from the company informing it that the employee had named it beneficiary of his plan and indicating the approximate amount that it would receive upon the employee’s retirement in 10 months. The letter did not inform the church of the employee’s right to change beneficiaries before that time. Church elders, anticipating the gift, authorized restoration work to the church building, making plans to pay for the work with the funds from the employee’s benefit program.

Six months later, the employee converted to a different religion and changed the beneficiary of his plan to his new church. When the employee retired, the company paid the benefit to his new church. His old church, which had paid for the restoration work on its completion, demanded payment of the benefit from the company. When payment was refused, the church sued the company.

Which party is likely to prevail?

(A) The church, because the interests of justice require it.

(B) The church, because its rights as third- party beneficiary had vested when it was informed in writing that it was the beneficiary.

(C) The company, because the agreement between the employee and the company allowed the employee to change the beneficiary of the benefit plan

(D) The company, because it had a duty to pay the employee’s new church as the named beneficiary of his plan.

A

So, A because there was estoppel.

(B) is incorrect because the church’s rights as a third-party beneficiary did not vest on receipt of the letter. In the usual case, an intended third-party beneficiary can prevent the contracting parties from rescinding or modifying the contract once his rights have vested. Vesting occurs when the beneficiary: (i) manifests assent to the promise in a manner invited or requested by the parties; (ii) brings suit to enforce the promise; or (iii) materially changes position in justifiable reliance on the promise. Merely being informed does not cause vesting; reliance is required

(A) The church will be able to recover against the insurance company because the interests of justice require it. Under the majority view, consideration is not necessary to make an agreement at least partially enforceable where the facts indicate that the promisor should be estopped from not performing. This is stating the concept of promissory estoppel without labeling it as such. Under the Second Restatement, a promise is enforceable to the extent necessary to prevent injustice if the promisor should reasonably expect the promise to induce action or forbearance and such action or forbearance is in fact induced. Here, the insurance company sent a letter to the church informing it that the employee had named the church beneficiary under his employee benefits program. The company did not warn the church that the employee had the right to change his beneficiary and should have reasonably expected that the church would rely on the promise in some way; it is not necessary in charitable contribution cases that the promisor know of a specific expenditure that the recipient made or is going to make.

How well did you know this?
1
Not at all
2
3
4
5
Perfectly
3
Q

A police officer went to the defendant’s house and placed him under arrest for operating an auto theft ring. As the defendant was being arrested, he told his wife, “You had better

call our lawyer; I don’t want to sign anything unless she’s with me.” The defendant was given Miranda warnings on the way to the police station. Meanwhile, the defendant’s lawyer called the station and told the desk sergeant that she was on her way and to have the defendant call her as soon as he arrived. The sergeant assured her that the defendant would be held without questioning for several hours until the district attorney arrived. When the defendant arrived at the station, the arresting officer and another officer immediately put the defendant in an interrogation room and questioned him about a bank robbery that had taken place two days ago. They did not inform him of the call from his lawyer, but he agreed to talk as long as he did not have to put anything in writing or sign anything without her okay. He made incrimi- nating statements about the robbery, and he was eventually indicted for that crime as well. Prior to trial on the robbery charge, the defendant’s lawyer moved to suppress the arresting officer’s testimony about the defendant’s statements.

How should the court rule?

(A) Deny the motion, because the questioning was about a different crime from the one for which the defendant was in custody.

(B) Deny the motion, because the defendant’s statements were made voluntarily after receiving Miranda warnings.

(C) Grant the motion, because the defendant was not informed that his lawyer was trying to see him, and his lawyer was misinformed that he would not be questioned right away.

(D) Grant the motion, because the defendant’s refusal to write or sign anything indicates that he did not knowingly and intelligently waive his right to the assistance of counsel.

A

(B) The defendant’s motion should be denied because his interrogation did not violate his Fifth Amendment right to counsel. At any time prior to or during interrogation, a suspect may invoke a Miranda (Fifth Amendment) right to counsel. However, the request must be unambiguous and specific. If the defendant agrees to answer questions orally, but requests the presence of counsel before making any written statements, the defendant’s oral statements are admissible. The defendant’s agreement to talk constitutes a voluntary and knowing waiver of the right to counsel, even if it could be argued that it indicates a misunderstanding of the evidentiary effect of oral statements.

How well did you know this?
1
Not at all
2
3
4
5
Perfectly
4
Q

To encourage minority business and foster pride in minority heritage, a state adopted legislation exempting magazines and other periodicals from the state’s receipts tax if 20% of the magazine is devoted to articles concerning minorities (a commission was set up to sample magazines to determine on a yearly basis whether they should be exempt). A publisher produced a sports magazine in the state that occasionally contained articles about minority athletes, but the commission determined that

the publisher’s magazine was not eligible for
the receipts tax exemption. After paying the tax assessed on her magazine, the publisher sued for a refund.

How will the court most likely rule?

(A) Against the publisher, because taxpayers do not have standing to challenge tax exemp- tions.

(B) Against the publisher, because the state has a compelling interest in encouraging minority business.

(C) In favor of the publisher, because the tax violates the Equal Protection Clause.

(D) In favor of the publisher, because the tax violates the First Amendment freedoms of speech and press.

A

(D) The court should rule in favor of the publisher because the tax exemption regulates speech based on its content in violation of the First Amendment. The freedom of the press is guaranteed by the First Amendment. As with other areas within the First Amendment, the freedom does not prohibit all government regulation of the press, but it does place limits on regulation. The press and broad- casting companies can be subject to general business regulations and taxes, but generally may not be singled out for a special tax. Moreover, a tax impacting on the press or a subpart of the press cannot be based on the content of the publication absent a compelling justification.

Although the state tax here appears to be a general receipts tax, the exemption is based on content, which means that the tax also is based on content (i.e., a publication is subject to the tax unless it contains . . . ). As discussed below, a compelling interest is not presented here, so the exemp-
tion is invalid and the tax should fail.

The Equal Protection Clause prohibits government discrimination absent a compelling interest, and laws that favor a minority are subject to the same strict scrutiny standard as laws that discriminate against a minority. However, the Supreme Court has found that remedying past discrimination against a minority—either by the government or by the public—is a compelling interest. Therefore, a government program favoring a minority will be upheld if it is narrowly drawn to remedy past discrimination. Here, we are not given any facts about past discrimination and so cannot decide whether the Equal Protection Clause has been violated.

How well did you know this?
1
Not at all
2
3
4
5
Perfectly
5
Q

A motorist was driving to a luncheon in a car that he knew did not have operating headlights. On the way there he was rear-ended by another driver who had been driving 20 m.p.h. over

the speed limit posted on that stretch of road. He suffered personal injuries and his car was extensively damaged. The jurisdiction makes it a misdemeanor to drive a vehicle that does not have operating headlights.

If the motorist brings an action against the other driver and the above facts are established, will he prevail?

(A) Yes, because the other driver violated the speeding statute, but the motorist’s damag- es will be reduced because of his violation of the headlight statute.

(B) Yes, because the other driver violated the speeding statute, and the motorist’s damages will not be reduced despite his violation of the headlight statute.

(C) No, because the motorist’s violation of the headlight statute constitutes negligence per se.

(D) No, because the motorist has not estab- lished that driving 20 m.p.h. over the speed limit created an unreasonable risk of injury to others.

A

(B) The motorist will recover all of his damages because the other driver’s violation of the statute constituted negligence per se. A clearly stated duty created by a criminal statute may replace the more general duty of care if the proponent of the statutory standard shows that (i) he is in a class intended to be protected by the statute, and (ii) the statute was designed to prevent the type of harm that was suffered. Here, the motorist can establish that the statutory standard regarding speeding should be applied against the other driver because the speed limit was posted, the motorist, as a fellow driver, is in the class intended to be protected by the statute, and it was designed to prevent accidents such as that which occurred.

(A) is incorrect because, while an applicable statute may establish plaintiff’s contributory negligence, the headlight statute does not apply here. Even though the statute was intended to protect drivers against cars being driven without headlights, it would be very difficult to show that it was designed to prevent rear-end collisions during the day, or that violation of the statute was a cause of the motorist’s injury

How well did you know this?
1
Not at all
2
3
4
5
Perfectly
6
Q

An owner purchased a parcel of property adjoining a five-foot-wide strip, which was a private right-of-way. Unsure where the exact boundaries of her property were located, the owner planted a garden on the five-foot right-of- way strip and enclosed it with a wire fence two weeks after taking up occupancy. The owner maintained the fence and garden for 20 years, at which time she removed the fence and smoothed out the ground where the garden had been located. Five years later, the owner entered into a written contract to sell the property to a buyer. The description in the contract included the five- foot strip. After research in the county recorder’s office, the buyer discovered that the strip was a private right-of-way when the owner purchased the property. After properly notifying the owner of the problem prior to closing, the buyer refused to tender the purchase money to the owner

when the closing day arrived. The owner sued the buyer for specific performance of the real estate sales contract. The jurisdiction’s statutory adverse possession period is 15 years.

Who will prevail?

(A) The buyer, because the owner failed to provide a marketable title.

(B) The buyer, because the owner surrendered her adverse possession rights when she removed the fence, as her possession was no longer open, notorious, and continuous.

(C) The buyer, because one may not adversely possess a right-of-way.

(D) Theowner,becausesheheldtheright-of- way for a longer time than the minimum required by the state adverse possession statute

A

(A) Absent a judgment in an action to quiet title or other tangible proof that title to the five-foot strip has actually been acquired, most jurisdictions would not consider the owner’s title marketable. All contracts for the sale of land contain, unless the contract expressly provides otherwise, an implied covenant by the seller that she will deliver to the buyer a marketable title at the date of closing. Marketability refers to freedom from the possibility of litigation concerning the title; title is marketable if a reasonably prudent buyer, ready and able to purchase, will accept it in the exercise of ordinary prudence. At times, sellers will rely on adverse possession to show that defects in title have been cleared. However, courts generally will not permit such reliance when proof of adverse possession rests only on oral evidence that will not be available to the buyer in the future. Here, title to the property described in the contract is unmarketable because the five-foot strip was a private right-of-way and not part of the owner’s record title. The owner’s adverse possession of

the strip will not be sufficient by itself to establish marketable title; there is no longer any physical evidence of the owner’s possession. Thus, at the least the owner must offer the buyer additional proof that the buyer can use to defend any lawsuit challenging title.

(B) is wrong because the owner removed the fence after she had acquired title by adverse possession. While that makes it more difficult for her to establish marketable title in selling the property, it does not affect the ownership rights she gained by adverse possession.

How well did you know this?
1
Not at all
2
3
4
5
Perfectly
7
Q

Question 14

A jogger found a stray dog in the park. She took the dog home with her and placed an ad in the paper to try to find the dog’s owner. Soon thereafter, the owner of the dog contacted the jogger. He came to the jogger’s home and identi- fied the dog as his. He offered to pay the jogger a $200 reward at the end of the week. The jogger thanked the dog owner but turned down the reward.

At the end of the week, however, the jogger changed her mind, so she called the dog owner and told him that she would like the reward after all. He refused to pay her, and she sues him for breach of contract.

What will the jogger recover?

(A) Nothing, because she rejected the dog owner’s offer.

(B) Nothing, because there was no consider- ation to support a contract.

(C) $200, because the technical defense of the Statute of Frauds will be overcome by the dog owner’s moral obligation to pay.

(D) $200, because the dog owner could not have revoked his offer until the end of the week, and he failed to do so before the jogger accepted.

A

(B) The jogger will recover nothing because her finding the lost dog occurred prior to the dog owner’s promise to pay the $200. An enforceable contract must be supported by consideration. Consideration consists of: (i) a bargained-for exchange between the parties; and (ii) an element of legal value to that which is bargained for. Legal value is present if the promisee has incurred a detriment (i.e., has done something she is under no legal obligation to do or has refrained from doing something that she has a legal right to do). To have a “bargained-for exchange,” the promise must induce the detriment, and the detriment must induce the promise. If something has already been given or performed before the promise is made, it will not satisfy the bargain requirement, because it was not given in exchange for the promise. Here, the jogger was under no legal obligation to return the dog to its owner. Thus, in doing so, she incurred a detriment. However, the jogger was not induced to so act by the dog owner’s promise to pay $200.

How well did you know this?
1
Not at all
2
3
4
5
Perfectly
8
Q

Two brothers who were certified public accountants worked together at a large accounting firm practicing their chosen profes- sion. The older brother was concerned about
his younger brother’s apparent inability to show up at his job by 9 a.m. each morning, sober and clear-eyed. One day, after the younger brother showed up late for work yet again, the older brother told him that if he would show up at the office sober and ready to work by 9 a.m. each morning for the next 10 months, he would pay him $15,000 at the end of that time. The younger brother accepted the offer and complied with its terms from that day forward. Nine months later, the older brother died unexpectedly. One month after that, the younger brother filed a claim with his brother’s estate for the $15,000.

Will the younger brother prevail in his claim?

(A) No, because he will be unable to prove the terms of the oral contract between him and his brother, because his brother is dead.

(B) No, because his brother’s offer to pay was terminated on his death.

(C) Yes, because he has performed under a valid contract, and thus his brother’s estate must now perform.

(D) Yes, because he changed his position for the worse in reliance on his brother’s promise, and thus his brother’s executor is estopped from denying that the contract existed.

A

(B) is incorrect because an offer will not be terminated by the death of the offeror if the offeror’s power to revoke is limited by law, such as in the case of a valid unilateral contract. Here, the younger brother has begun performance, making the offer irrevocable during the time he was given to complete performance

(C) The younger brother will prevail because he has performed under a valid contract. He entered into and performed a valid unilateral contract with his brother, who offered to give him $15,000 if he showed up at the office sober and ready to work by 9 o’clock each morning for the next 10 months. He accepted by fully performing; his giving up the right to do something that he had a legal right to do constitutes valid consideration. Because the younger brother fully performed his duties under the contract, the older brother’s estate is bound to perform his duties and must now pay him.

How well did you know this?
1
Not at all
2
3
4
5
Perfectly
9
Q

[example where being more literal would have helped]

On the last play of a playoff football game,
a game-winning touchdown was nullified by a questionable penalty called by the referee. To register her displeasure but without intending
to hit anyone, a fan sitting in the stands threw a bottle onto the field that just missed the head of the referee, who was looking in the other direc- tion and did not see the bottle being thrown. The fan was charged with assault.

Should the fan be convicted?

(A) Yes, because throwing the bottle was a substantial step towards commission of a battery.

(B) No, because the referee did not see the bottle.

(C) No, because the fan did not intend to hit anyone.

(D) No, because the referee did not see the bottle, nor did the fan intend to hit anyone.

A

(D) The fan should not be convicted under either definition of assault. Criminal assault is either: (i)
an attempt to commit a battery, or (ii) the intentional creation, other than by mere words, of a reasonable apprehension in the mind of the victim of imminent bodily harm. In the instant case, the fan did not intend to hit anyone. Thus, the fan’s actions do not constitute an attempt to commit a battery, which would require a specific intent to bring about bodily injury or an offensive touching. In addition, the fact that the referee did not see the bottle being thrown at him means that the defendant did not create in the referee a reasonable apprehension of imminent bodily harm. Consequently, the fan is not likely to be convicted of either type of assault. This also makes (B) and (C) incorrect, in that each answer considers only one type of assault

How well did you know this?
1
Not at all
2
3
4
5
Perfectly
10
Q

In January, an owner executed and deliv- ered a mortgage on her property to a bank to secure a $50,000 loan. Due to a clerical error, the mortgage was not recorded at that time. On February 15, the owner entered into a contract to sell the property to a buyer for $150,000.
On February 16, the owner took out a $30,000 mortgage on the property with a finance company. The finance company promptly
and properly recorded its mortgage. Knowing nothing about either of the mortgages, the buyer closed on the property on April 1, tendering $150,000 to the owner. The owner gave the buyer a warranty deed to the property. On April 3, the bank discovered its error and properly recorded its mortgage that same day. The buyer recorded his deed to the property on April 6.

A statute of the jurisdiction in which the property is located provides: “No conveyance or mortgage of real property shall be valid against a subsequent purchaser for value and without notice whose conveyance is first recorded.” The bank brings an appropriate action to determine the status of its mortgage on the property.

What should be the court’s determination?

(A) The buyer holds the property subject to both mortgages, and the bank’s mortgage is subordinate to the finance company’s mortgage.

(B) The buyer holds the property subject to both mortgages, and the bank’s mortgage is superior to the finance company’s mortgage.

(C) The buyer holds the property subject only to the finance company’s mortgage.

(D) The buyer holds the property subject only to the bank’s mortgage.

A

(A) The buyer holds the property subject to both mortgages, and the bank’s mortgage is subordinate to the finance company’s mortgage. The jurisdiction’s recording act is a race-notice statute. Under this statute, a bona fide purchaser is protected only if he records before the prior transferee or mortgagee records. Here, the buyer had record notice of the finance company’s mortgage, so he was not a bona fide purchaser protected by the recording statute as to that mortgage. As to the bank’s mortgage, the buyer was a bona fide purchaser because he had no notice of that mortgage executed by the owner, but he did not record until after that mortgage was recorded. Hence, the buyer holds the property subject to both mortgages. The bank’s mortgage is subordinate to the finance company’s mortgage because mortgagees for value are treated as “purchasers” under the recording statutes, and the finance company executed its mortgage without notice of the bank’s prior mortgage and recorded it before the bank recorded its mortgage. Thus, the finance compa- ny’s mortgage has priority over the bank’s mortgage,

How well did you know this?
1
Not at all
2
3
4
5
Perfectly
11
Q

A developer contracted with a general contractor to build an office building, and completion of the building was two years late. The developer filed a breach of contract action in federal district court against the general contractor, seeking damages caused by the delay. The general contractor filed a third-party claim against a major subcontractor, claiming that the subcontractor caused any delay and should be liable to the general contractor for anything the general contractor has to pay the developer. The subcontractor believes that the developer inter- fered with the subcontract and that the devel- oper’s interference caused not only the delay but also substantial cost overruns for the subcon- tractor.

May the subcontractor assert a claim in the pending action against the developer seeking payment for the cost overruns?

(A) No, because the subcontractor is a third- party defendant and may not assert claims against the original plaintiff.

(B) No, because the subcontractor’s claim does not seek indemnity for its liability to the general contractor, so the subcontractor may not assert this claim against the devel- oper as an impleader claim.

(C) Yes, because the subcontractor and the developer are already parties to the action, and the subcontractor’s claim arises from the same transaction or occurrence as the developer’s original claim, so the subcon- tractor will be barred from asserting the claim in an independent action.

(D) Yes, because the subcontractor’s claim against the developer arises from the same transaction or occurrence as the developer’s original claim, but the subcontractor may assert the claim in an independent action if it prefers.

A

[did now know this]

(D) The subcontractor may assert a claim against the developer in the pending action. A third-party defendant may assert a claim against the plaintiff if the claim arises out of the same transaction or occurrence as the plaintiff’s original claim.

(C) is incorrect because a third- party defendant’s claim against the plaintiff is not compulsory.

How well did you know this?
1
Not at all
2
3
4
5
Perfectly
12
Q

The defendant became very intoxicated one night. As he was staggering home, he came upon a construction site in which several large pieces of heavy equipment were parked. Having had heavy equipment training in the military, the defendant decided it would be fun to rearrange all the machines so that the operators would
be very surprised when they returned to work the next day. He started up the largest piece of heavy equipment and drove it toward the edge
of the site, but because he was so intoxicated,
he lost control of it, and it rumbled out into the street, weaved along for about a quarter mile, and then crashed into a house, flattening it. In this jurisdiction, it is a misdemeanor to tamper with heavy equipment on a construction site. The defendant is prosecuted on the tampering charge, as well as for reckless damage of the house.

Should he be convicted of the reckless damage charge?

(A) Yes, because he was tampering with heavy equipment on a construction site, in viola- tion of law, when he damaged the house.

(B) Yes, because he was intoxicated while driving a huge piece of earthmoving equip- ment.

(C) No, because at most he could be found guilty of criminal negligence.

(D) No, because he must have been aware that his conduct would cause damage to the house in order to be found guilty of reckless damage.

A

[recklessness is not negligence]

B) The defendant should be convicted because he was intoxicated when he damaged the property. The defendant is being charged with reckless damage to property. A person acts recklessly when he consciously disregards a substantial or unjustifiable risk that a prohibited result will follow and this disregard constitutes a gross deviation from the standard of reasonable care. Driving earth- moving equipment while intoxicated would be considered to be reckless because of the great potential for destruction arising from the huge size and power of the equipment.

How well did you know this?
1
Not at all
2
3
4
5
Perfectly
13
Q

[always saw this around]

During the defendant’s prosecution for robbery, the prosecutor asks the court to take judicial notice of the fact that at that latitude, the sun is still up at 5:30 p.m. on June 21. The court so finds.

What is the effect of the court’s action?

(A) The burden of persuasion is now on the defendant to prove otherwise as to the fact judicially noticed.

(B) The fact judicially noticed is established beyond a reasonable doubt.

(C) The prosecutor’s burden of producing evidence on the fact judicially noticed is satisfied.

(D) The fact judicially noticed is conclusively established.

A

(C) Judicial notice operates as a substitute for proof as to facts that are matters of common knowledge in the community or are capable of certain verification through easily accessible, well-established sources. When a court takes judicial notice of a fact under the federal rules in a criminal case,
the jury may, but is not required to, accept the fact noticed; thus, its effect is only to relieve the prosecutor of her burden of producing evidence on that fact.

How well did you know this?
1
Not at all
2
3
4
5
Perfectly
14
Q

[look at definition of looking into people’s pockets]

Based on a tip from a reliable informant
that an attorney was illegally selling automatic weapons and ammunition from his storefront office, the police obtained a warrant to search for weapons at the office. When they arrived at the building, they saw a client exiting the attorney’s office and placing what appeared to be a weapon inside his jacket. The police stopped the client on the street and an officer patted down his outer clothing. The officer felt no weapon but did feel a bag with several small tube-shaped objects
in it. She immediately placed the client under arrest. The contents of the bag were later deter- mined to be marijuana cigarettes.

Prior to trial on the narcotics charge, the client sought to suppress introduction of the marijuana as evidence. The arresting officer testified at
the suppression hearing that, based on her long experience as a narcotics officer, she concluded immediately that the bag contained marijuana cigarettes when she first touched it.

If the officer’s testimony is believed, how should the court rule on the motion to suppress the marijuana evidence?

(A) Deny it, because the search was incident to a lawful arrest.

(B) Deny it, because the police had a reasonable suspicion that the client might be armed and dangerous.

(C) Grant it, because the scope of an officer’s patdown during an investigatory detention is limited to a search for weapons.

(D) Grant it, because the search warrant did not authorize the police to search the client despite the fact that he was just present at the place to be searched.

A

(B) The client’s motion should be denied because the seizure of the marijuana was properly within
the scope of the stop and frisk. A police officer may stop a person without probable cause for arrest if she has an articulable and reasonable suspicion of criminal activity. [Terry v. Ohio (1968)] In such circumstances, if the officer reasonably believes that the person may be armed and dangerous, she may conduct a protective frisk. The scope of the frisk is limited to a patdown of the outer clothing for concealed instruments of assault, but the officer may reach into the suspect’s clothing and seize any item that the officer reasonably believes, based on its “plain feel,” is a weapon or contraband. [Minnesota v. Dickerson (1993)]

Here, the officer believed that the client put a weapon in his jacket as he was leaving a place where weapons and ammunition were being sold illegally; thus, she had reasonable grounds to conduct both a stop and a frisk. If the court accepts the officer’s testimony that she instantly recognized the marijuana cigarettes based on the patdown only without any further conduct, they were properly seized and can be admitted into evidence.

How well did you know this?
1
Not at all
2
3
4
5
Perfectly
15
Q

[type of notice]

An owner of three acres of lakefront property subdivided it and sold two acres to a buyer, retaining the one acre actually fronting on
the lake. The deed for the two acres expressly included an easement over the westernmost

30 feet of the one-acre parcel retained by the owner for access to the lake. The buyer recorded his deed in the county recorder’s office, which maintained an alphabetical grantor-grantee index only. Fifteen years later, the owner died, leaving the one-acre parcel to his wife. She sold it to a developer that planned to build condominiums. A month later, the buyer died, and his two acres passed by will to his nephew. Three weeks after taking title to the property, the nephew visited the property and discovered that the developer had erected a chain link fence all along the boundary between the nephew’s land and the acre of lakefront land. The nephew brings an action to enjoin the developer from obstructing his easement across the acre of lakefront property.

Which of the following best describes why the nephew should prevail in this litigation?

(A) Because the developer and the nephew
can trace their predecessors in interest to a common grantor whose covenants run with the land, the developer is estopped from interfering with the nephew’s use of the easement.

(B) The nephew’s easement is a legal interest that the developer has record notice of, even though there is no tract index.

(C) Because there is no tract index, the devel- oper was under an obligation to deter- mine the riparian rights of any adjacent landowners before erecting the chain link fence.

(D) The nephew’s easement is a legal interest that attaches not just to a legal estate but to the land itself and, running with the land, it binds successive owners of the servient estate whether or not they have notice of it.

A

(B) The nephew should prevail because his interest is a legal interest in the property and could have been discovered by the developer in the grantor-grantee index. The owner granted the buyer
an easement by express grant. The easement was properly recorded with the buyer’s deed,
and because it contained no limitation, it is perpetual. The easement here is appurtenant (i.e.,

one benefiting the holder of the easement), because it benefits the buyer’s land (the dominant tenement) and burdens the owner’s land (the servient tenement). Where there is an easement appurtenant, it passes with a transfer of the dominant tenement, even though it is an interest in
the servient tenement. Thus, the buyer’s easement passed to his nephew. Because the easement
is perpetual, it is binding on all of the owner’s subsequent transferees regardless of whether the conveyance refers to the easement, as long as the transferees have notice of it. Many courts will find record or constructive notice here because the nephew’s property is adjacent to the devel- oper’s property, is deeded from a common grantor (the owner), and includes the easement in the original deed from the owner.

(C) is incorrect because it is untrue. Purchasers of land are subject to easements in their chains of title regardless of the type of recording index, because they are deemed to have constructive notice of any easement that was recorded. There is no independent duty to discover riparian rights when there is no tract index. The nephew can enforce the easement here only because it was in the chain of title of the developer’s property.

How well did you know this?
1
Not at all
2
3
4
5
Perfectly
16
Q

The accused was driving his beat-up old car along a narrow road when he was passed by the victim in her new car. The victim’s daughter was lying down in the back seat and could not be seen. The accused sped up, drew even with the victim, and repeatedly rammed his car
into the side of the victim’s car. After several collisions, the victim was forced off the road, rolling down a cliff for several yards. Due to the rolling, both the victim and her daughter were severely injured. The accused was charged with attempted murder of both of them. At his trial, he testifies that he was angry because of the cavalier way the victim passed him in her new car, and that his only intent in smashing into her car was to scratch and dent it so that she would not be so haughty in the future.

Assuming that the jury believes this testi- mony, of whom may the accused be convicted of attempted murder?

(A) The victim.

(B) The victim’s daughter.

(C) Both the victim and her daughter.

(D) Neitherthevictimnorherdaughter.

A

(D) The accused may not be convicted of attempted murder because he lacked the necessary intent. A criminal attempt consists of (i) a specific intent to commit the crime; and (ii) an overt act in furtherance of that intent. In other words, the defendant must have the intent to perform an act and obtain a result that would constitute the crime charged if achieved. Regardless of the intent required for the completed offense, an attempt always requires a specific intent. Thus, attempted murder requires the specific intent to kill another person, even though the mens rea for murder itself does not require a specific intent—had the victim or her daughter died, the accused could be convicted of murder because malice aforethought can be established here by awareness of an unjustifiably high risk to human life (i.e., “abandoned and malignant heart”). However, the accused did not have the intent to kill either victim, so he lacked the intent necessary for attempt.

How well did you know this?
1
Not at all
2
3
4
5
Perfectly
17
Q

At the trial of the plaintiff’s breach of contract action against the defendant, the plaintiff called her accountant as a witness to testify about the difference in gross sales, gross income, and

net profit caused by the defendant’s failure to supply the promised quantity of ice cream to the plaintiff’s ice cream shop. When the plaintiff’s attorney asked the accountant to state the gross income figures for the year prior to formation

of the contract between the plaintiff and the defendant, the accountant replied that he could not remember the exact amounts. The plaintiff’s counsel then handed the accountant a copy of the federal tax return submitted by the plaintiff for that year, and asked him to read it. Counsel then asked, “Now that you have read the tax return, can you remember what the gross income of the plaintiff’s ice cream shop was for the relevant period?” The defendant’s counsel objects.

How should the court rule?

(A) Sustained, because the plaintiff’s counsel is seeking to elicit testimony based on inad- missible hearsay.

(B) Sustained, because the accountant’s testi- mony is not the best evidence.

(C) Overruled, because the accountant’s hearsay testimony is admissible as a past recollection recorded.

(D) Overruled,becausetheaccountant’stesti- mony is admissible evidence relating to the plaintiff’s damages.

A

D) The objection should be overruled because the accountant’s testimony is admissible. As the plain- tiff’s accountant, he has personal knowledge of the relevant financial information, and so may testify. Although the accountant indicated that he could not remember the plaintiff’s income, the rules of evidence allow a witness’s recollection to be refreshed by just about anything. The witness may not read from the writing while he testifies; it is used solely to jog his memory. While the opposing counsel is allowed to examine the item being used to refresh the witness’s testimony and may cross-examine the witness about it, he may not object to it

(B) is incorrect because the best evidence rule requires only that when the contents of a writing are sought to be proved, the writing itself should be entered into evidence, if it is available. Here, the contents of the tax return are not being entered into evidence; rather, the accountant is merely using the tax return to refresh his memory. Therefore, the best evidence rule does not apply.

How well did you know this?
1
Not at all
2
3
4
5
Perfectly
18
Q

[interesting mortgage question]

A photographer borrowed $100,000 from
a bank, secured by a mortgage on his home,
to build a studio and darkroom in the home. The bank properly recorded the mortgage.
After completing this project, the photographer decided to remodel his kitchen and borrowed $25,000 from a lending company, also securing the loan with a mortgage on his home. The lending company did not record its mortgage. After the remodeling was complete, the photog- rapher borrowed $15,000 from an investor, secured by a mortgage on his home, to redo his in-ground pool. Learning of this transaction, the lending company raced to the recording office and recorded its mortgage. The next day, the investor recorded its mortgage.

A few months later, the photographer defaulted on all three mortgages, having not made any principal payments. The lending company brought a foreclosure action, joining the investor in the proceeding. The foreclosure sale resulted in $150,000 in proceeds after all expenses and fees were paid. A statute of the jurisdiction in which the photographer’s home is located provides: “Any conveyance of an interest in land shall not be valid against any subsequent purchaser for value, without notice thereof, unless the conveyance is recorded.”

Which of the following statements is true?

(A) The bank is entitled to $100,000 of the foreclosure proceeds, the lending com- pany is entitled to $25,000 of the proceeds, the investor is entitled to $15,000 of the proceeds, and the buyer at the foreclosure sale is entitled to the remaining $10,000 in proceeds.

(B) The buyer at the foreclosure sale will take the home subject to the bank’s mortgage, the lending company is entitled to $25,000 of the proceeds, the investor is entitled to $15,000 of the proceeds, and the photogra- pher is entitled to the remaining $110,000 in proceeds.

(C) The buyer at the foreclosure sale will take the home subject to the bank’s and the investor’s mortgages, the lending company is entitled to $25,000 of the proceeds,

and the photographer is entitled to the remaining $125,000 in proceeds.

(D) The buyer at the foreclosure sale will take the home subject to the bank’s and the investor’s mortgages, the lending company is entitled to $25,000 of the proceeds, and the buyer at the foreclosure sale is entitled to the remaining $125,000 in proceeds.

A

(B) is wrong because the investor’s mortgage is also senior to the lending company’s, and as a result, the buyer also takes subject to the investor’s mortgage. Under a notice statute, which this jurisdiction has, a subsequent bona fide purchaser prevails over a

prior grantee who fails to record. The important fact under a notice statute is that the subsequent purchaser had no actual or constructive notice at the time of the conveyance (or mortgage), not at the time of recording. Mortgagees for value are treated as “purchasers.” Here, when the mortgage on the property was granted to the investor, it had neither actual nor constructive notice of the mortgage given to the lending company. The fact that the lending company recorded its mortgage first is irrelevant. Thus, the investor was a bona fide purchaser and would be entitled to protection under the statute.

(C) The buyer at the foreclosure sale will take the home subject to the bank’s and the investor’s mortgages, the lending company is entitled to $25,000 of the proceeds, and the photographer is entitled to the remaining $125,000 in proceeds. When an interest is foreclosed, after the expenses and fees are paid, the proceeds of the sale are first used to pay the principal and accrued interest on the loan that was foreclosed, next to pay off any junior liens, and finally any remaining proceeds are distributed to the mortgagor. (A) is wrong because the bank’s interest, an interest senior to the lending company’s, is not affected by the foreclosure. As a senior interest, the bank was not a necessary party to the foreclosure action and did not need to be named in the foreclo- sure action. Thus, the bank is not entitled to a share of the proceeds, and its lien continues on the property in the buyer’s hands

How well did you know this?
1
Not at all
2
3
4
5
Perfectly
19
Q

After filing a complaint in a federal district court, a plaintiff retained an appropriate private process server to serve the summons and complaint on the defendant, an individual who resides in a mansion in the state in which the court is located. In his first attempt to serve process, the process server knocked on the mansion’s main door, and the door was answered by the mansion’s caretaker, who resides in
the mansion. The process server learned that
the defendant resides in the house September through May of each year, but lives in another home during the other months. Given that it was October at the time, the defendant was currently residing there, but not present at that particular time. The process server left the summons and complaint with the caretaker, and he mailed copies of the summons and complaint to the defendant’s mansion address and to the defen- dant’s other home. The defendant’s attorney
filed a motion to dismiss the action for improper service of process.

Should the court grant the motion to dismiss?

(A) No, because the process server mailed cop- ies of the summons and complaint to the defendant’s homes.

(B) No, because the caretaker resides at the mansion.

(C) Yes, because service was not made on a member of the defendant’s family.

(D) Yes, because the mansion is not the defen- dant’s year-round permanent home.

A

(B) The court should not grant the motion to dismiss. Federal Rule of Civil Procedure 4 provides that service may be made on an individual in the United States by leaving a copy of the summons and complaint at the individual’s dwelling or usual place of abode with someone of suitable age and discretion who resides there

How well did you know this?
1
Not at all
2
3
4
5
Perfectly
20
Q

A brother and a sister were arrested on the federal charge of tax evasion in connection with the family business. Prior to trial, the prosecutor told the sister that he believed he could get her sentence reduced to probation if she pleaded guilty to a lesser charge and agreed to testify against her brother; the sister reluctantly agreed. During the jury trial, the sister is called by the prosecution. On cross-examination, the defense attorney brings out the fact that the sister was arrested on the same charge. The attorney then asks her whether it is true that after her arrest, the prosecutor told her that if she testifies against her brother her sentence can be reduced to probation. The prosecutor objects.

How should the court rule on the objection?

(A) Sustained, because it is against public policy to reveal information about plea bargains to a jury.

(B) Sustained, because it calls for hearsay.

(C) Overruled, because the question goes to bias or interest.

(D) Overruled,becausethesisterwaivedthe attorney-client privilege by testifying.

A

Under Federal Rule 410, withdrawn guilty pleas, pleas of nolo contendere, offers to plead guilty, and evidence of statements made in negotiating such pleas are inadmissible against the defendant who made the plea or was a participant in the plea discussions. This rule does not apply in this case because it does not apply to accepted guilty pleas, and the sister is not the defendant. The rule applies only to offers and withdrawn pleas. After the plea is accepted, it is admissible.

How well did you know this?
1
Not at all
2
3
4
5
Perfectly
21
Q

A buyer of a new car owed the car dealer-
ship where she purchased the vehicle $1,000 on a promissory note that was due on December
30. The buyer determined that she would be unable to pay the note on its due date, and she informed the owner of the dealership of that fact. The owner told her that she would not have to pay the debt if she bought him four tickets to a popular concert on January 15 that had been sold out for weeks, because she worked as publicist for the concert venue. She agreed to do so, and the parties memorialized their agreement in a signed writing on December 18. On January 2, the dealership filed suit against the car buyer for failure to pay the $1,000 promissory note, before the car buyer had secured the concert tickets for the owner.

May the car buyer have this action enjoined by introducing evidence of the December 18 agree- ment?

(A) Yes, because the December 18 agreement between the parties suspended the car buyer’s obligation on the promissory note.

(B) Yes, because the December 18 agreement between the parties discharged the car buyer’s obligation on the promissory note.

(C) No, because the car buyer has not yet bought the concert tickets in reliance on the owner’s promise to extinguish the debt.

(D) No, because the car buyer’s only remedy is to sue for damages for breach of the December 18 agreement.

A

Remember, when breached can sue either or

(A) The car buyer may enjoin the dealership’s action because the dealership currently does not have the right to enforce the promissory note. A valid accord, taken alone, does not discharge the prior contract. It merely suspends the right to enforce it in accordance with the terms of the accord contract. The performance of the accord agreement, which is called satisfaction, discharges

not only the accord agreement but the original contract as well. Where the accord agreement is breached by the creditor by suing on the original contract, as is the case here, the debtor may seek to have the action enjoined by raising the accord agreement as an equitable defense. (B) is incorrect because the accord agreement does not discharge the original obligation, it only suspends it. Hence, if the car buyer were to breach the accord agreement, the dealership could sue on either the original promissory note obligation or the accord agreement.

22
Q

A security officer employed by a mall was patrolling the mall parking lot which had suffered numerous thefts from cars when
she heard a car alarm go off. She then saw a teenager stand up from behind the car. She immediately stopped the teenager and asked him what he was doing behind the car, and he said he was tying his shoe. Reasonably suspecting that he may have been trying to break into the car, she asked him to wait in the back seat of her car. The teenager complied, and waited in the back of the car while the security officer ate her lunch in the front seat and made a variety of personal calls. The teenager was humiliated because several of his friends and their parents, as well as some neighbors, saw him sitting in the security officer’s car. Finally, after about an hour, the officer let the teenager go and advised him not to loiter in the parking lot anymore.

If the teenager brings a false imprisonment action against the mall, will he be able to recover for the humiliation that he felt on sitting in the security officer’s car in view of his friends and neighbors?

(A) No, humiliation is not actionable.

(B) No, because the security officer reasonably suspected that he was trying to break into a car.

(C) Yes, if the jury determines that the security officer’s conduct was extreme and outra- geous.

(D) Yes, because he was falsely imprisoned.

A

(D) The teenager will be able to recover for his humiliation because he was falsely imprisoned. All of the elements of a prima facie case for false imprisonment are present in these facts: an act

or omission by defendant that confined or restrained plaintiff to a bounded area, intent by the defendant to do so, and causation. Here, the confinement was brought about by the invalid use of legal authority by the security officer. The mall cannot avail itself of the shopkeepers’ privilege for detaining a suspected shoplifter because the detention must be for only a reasonable period
of time for the purpose of making an investigation, and here the hour-long detention clearly was unreasonable given the officer’s failure to even make an investigation during the detention. On proof of the prima facie case, the plaintiff can recover all foreseeable damages that arise from the tort. Humiliation is a foreseeable consequence of a false arrest, so the teenager will be able to recover

23
Q

In a civil action tried to a jury, the defendant objected to the introduction by the plaintiff of certain evidence without the judge’s first making a preliminary ruling on the admissibility of the evidence.

For which evidence is the defendant’s objec- tion not appropriate?

(A) Opinion testimony regarding the structural integrity of a building by an engineer called by the plaintiff, without a preliminary de- termination by the judge that the engineer is an expert.

(B) Hospital records pertaining to the plaintiff offered by the plaintiff, without a prelimi- nary determination by the judge that they were made as a regular activity of the hospital staff.

(C) Contract negotiations between the plaintiff and a third party, without a preliminary determination by the judge that the third party was the defendant’s agent.

(D) A paramedic’s testimony that the plaintiff’s wife, before she died, said that the defen- dant’s car went through a red light before hitting her, without a preliminary determi- nation by the judge that she made the state- ment under a sense of impending death.

A

So, if you are lost, this question was talking about the fact that issues of fact are determined by a jury and questions of law are determined by the judge.

(C) The defendant’s objection to the contract negotiations is not appropriate because whether an agency relationship existed is determined by the jury. The Federal Rules of Evidence distinguish preliminary facts to be decided by the jury, which determine whether the offered evidence is relevant to the issues in the case, from preliminary facts to be decided by the judge, which determines whether the offered evidence is competent to be admitted at all. Whether an agency relationship existed between the defendant and a third party is a question of fact to be decided by the jury; if the jury decides that the third party was not the defendant’s agent, it will disregard as irrelevant the evidence of contract negotiations undertaken by the third party. While the judge must find that the proponent of the contract negotiations has introduced enough evidence to allow the jury to find that an agency relationship existed, the ultimate determination of agency rests with the jury

24
Q

Concerned about the rising death toll on the state’s highways, a state legislature enacted
a statute providing for a summary one-year suspension of the driver’s license of any person convicted of three speeding violations within a 12-month period. The statute provided that an administrative hearing is immediately available upon request. However, that hearing is limited to a determination of whether the licensee is the same person who was convicted of the speeding violations.

A driver received three speeding citations in a three-week period and was convicted of all three charges. Her license was promptly suspended under the authority of the state statute. Without first seeking an administrative hearing, the driver files a suit in federal district court challenging the constitutionality of the statute.

Should the court uphold the constitutionality of the state law?

(A) Yes, because driving an automobile on the state’s highways is a privilege and not a right.

(B) Yes, because the state’s interest in promptly removing unsafe drivers from its roads outweighs the driver’s right to a prior hearing under these circumstances.

(C) No, because the law creates an irrebuttable presumption that all drivers falling within the ambit of the statute are unsafe.

(D) No, as a denial of due process without a prior hearing

A

(B) The state law is valid because the prior judicial determinations that the driver violated the speeding laws satisfy the procedural due process requirements of the Fourteenth Amendment. Under the Due Process Clause of the Fourteenth Amendment, the state must provide some fair process or procedure before it may deprive a person of “life, liberty, or property.” Fair proce- dure at a minimum requires an opportunity to present objections to the proposed action to a fair, neutral decisionmaker. Whether a prior evidentiary hearing is required and the extent of procedural requirements is determined by weighing (i) the importance of the individual interest involved, (ii) the value of specific procedural safeguards to that interest, and (iii) the govern- mental interest in fiscal and administrative efficiency.

25
Q

What does in mean to plead above 75000 in a diversity case?

A

It means that there must be a legally tenable possibility that recovery will exceed $75,000

26
Q

A plaintiff brought suit against a manufac- turing company, seeking to recover for damages he suffered when his car’s engine burst into flames following the use of an engine additive made by the company. The plaintiff contends that the manufacturing company was negligent and in breach of warranty.

An automobile engineer sat in court while the plaintiff testified to the events concerning the engine fire. The plaintiff’s testimony was not challenged or rebutted. The plaintiff calls the engineer to the stand and asks him whether, based on the plaintiff’s prior testimony, it was possible for a car engine to burst into flames as it did.

Would the engineer’s testimony be admis- sible?

(A) No, because the engineer’s opinion was not elicited by means of a hypothetical ques- tion.

(B) No, because the engineer was in the court while the plaintiff testified concerning the engine fire.

(C) Yes, because the engineer was in the court while the plaintiff testified concerning the engine fire.

(D) Yes, as long as the engineer’s opinion is based only on admissible evidence.

A

(C) The engineer’s testimony is admissible because it is based on knowledge gained by him at trial. Facts or data upon which expert opinions are based may be derived from presentation at trial. One acceptable method of doing this is to have the expert attend the trial and hear testimony estab- lishing the facts. Thus, (B) is incorrect. (A) is incorrect. Under the Federal Rules, a hypothetical question is not required to elicit an expert’s opinion. (D) is incorrect because an expert may also base his opinion on facts supplied to him outside the courtroom, including types of facts not admissible into evidence, as long as they are reasonably relied on by experts in the field.

27
Q

A large-scale bakery in the South entered
into a written contract with a commercial apple orchard in the upper Midwest to purchase 200 bushels of apples at a cost of $8 per bushel. The contract provided that the apple orchard would deliver the apples “F.O.B. Louisville Railroad Depot,” where the apples would be loaded onto a train headed south. The orchard assigned all of its rights under the contract to a large produce distributor which, in turn, hired a trucking company to deliver the apples to Louisville. En route to Louisville, the truck skidded off the road due to inclement weather and overturned, and the apples were destroyed. The bakery brought suit against the apple orchard for breach of contract.

What will be the probable outcome of the litigation?

(A) The bakery will lose.

(B) The bakery will recover the amount neces- sary to replace the destroyed apples, over the contract price.

(C) The bakery will recover the full contract price.

(D) The bakery will be able to compel specific performance of the contract.

A

[still do not fully get this one]

(B) If the bakery brings an action against the apple orchard, the bakery will be able to recover the costs of replacing the destroyed apples because the apple orchard remained liable on the assigned contract and it had the risk of loss. Although most contractual duties may be assigned—unless they are personal—and the obligee must accept performance from the delegate, the delegating party (delegator) remains liable on his obligation. Thus, an assignment of a contract that includes a delegation of duties does not relieve the assignor from its duty to perform. Here, the bakery did not receive the performance that was due (the apples), so it could sue the apple orchard to recover for the breach. When a nonbreaching buyer does not receive the contracted goods, it has several options: it can cancel the contract and recover any incidental damages, or it can purchase replace- ment goods and sue for the cost of replacement—“cover.” Damages under the latter option are measured by the difference between the contract price and the amount the buyer actually has

to pay for the replacement goods. Thus, (B) is correct and (A) is incorrect. Note that (A) would have been correct if UCC section 2-613 were applicable, because it provides for avoidance of the contract when goods are lost without fault of either party before risk of loss passes to the buyer. However, that section applies only when particular goods are identified to the contract when the contract was made; here, there is no designation of specific bushels of apples until shipment. (C) is not a proper measure of damages unless the bakery had already paid for the apples and wished to cancel (and the facts do not indicate this to be the case), because the contract price may not be enough to purchase replacement goods if the price of apples has risen, and would be too much

if the price has dropped

28
Q

During a nationwide trucker’s strike, striking drivers committed repeated acts of violence against independent truckers and railroad shipments that had replaced truck transportation. This prompted Congress to enact an emergency measure directing the President to dispatch United States Army troops to specified cities and rail and highway locations to preserve order and ensure the continued flow of commerce.

Is this enactment constitutional?

(A) No, because it infringes on the President’s authority to faithfully execute the laws of the United States.

(B) No, because it infringes on the President’s authority as Commander in Chief of the armed forces.

(C) Yes, under Congress’s power to regulate commerce.

(D) Yes, under Congress’s power to raise and support the armed forces.

A

Congress can support but not control armies

(D) is incorrect because the enactment does not appropriate money to support the armed forces, but seeks to control their activities.

(B) Although the President has no power to declare war, Article II, Section 2 makes the President commander in chief of the military, which affords the President extensive power to deploy military forces against any enemy, foreign or domestic. Congress lacks such power. Therefore, (B) is correct

29
Q

Is getting money enough for false pretenses?

A

YUP you get title to it.

30
Q

So, if employer is negligent in hiring. Employee, outside the scope of employment then hurts someone. Employer liable?

A

YUP, for negligent hiring

31
Q

The defendant was charged with embezzling $1 million from his employer, a bank, by trans- ferring the funds to a secret offshore account
in the bank’s name. Only the defendant and the bank’s vice president were authorized to draw funds from the account. The defendant testified that he had wired $1 million to the account but had done so at the direction of the bank’s vice president. The defendant stated under oath that he had no intent to embezzle bank funds. The government’s cross-examination of the defendant concentrated exclusively on his relationship and conversations with the vice president, who has committed suicide.

The defense now seeks to call a second witness, who is prepared to testify that he had worked with the defendant for 10 years and that the defendant had a reputation in both the business and general communities as being a very honest person.

Is the witness’s testimony admissible?

(A) Yes, because a defendant has a constitu- tional right to call witnesses on his own behalf.

(B) Yes, to help show that the defendant did not embezzle funds.

(C) No, because it is inadmissible character evidence.

(D) No, because you cannot bolster the credibility of your own witness unless the credibility of the witness has been attacked

A

(D) is wrong. It is true that a party may not present evidence to bolster the credibility of his own witness until the witness has been impeached. However, if the evidence of good character is offered for any other legitimate purpose, the fact that it also helps bolster the credibility of the witness will never result in the exclusion of the evidence. Here the evidence is admissible to show that the defendant may not have embezzled funds; hence, the rule excluding evidence offered to bolster credibility would not apply.

(B) The witness’s testimony is admissible. Under Federal Rule 404(a), a criminal defendant is allowed to present relevant character evidence to help establish that he may not have committed the crime charged. Thus, evidence that the defendant had a reputation for being an honest person would be admissible to show that he might not have embezzled funds because embezzlement is a crime of dishonesty.

32
Q

A masked gunman held up a convenience store. Due to the poor quality of the surveil- lance recording, it was very difficult to identify the masked gunman. Nonetheless, the defendant was arrested and charged with the robbery.
At the preliminary hearing, the magistrate, on seeing the poor quality of the tape, determined that there was not probable cause to prosecute the defendant. After that, the county prosecutor presented the case to a grand jury, but the grand jury refused to indict the defendant. After waiting a couple of months, the prosecutor presented the case to a different grand jury.
The grand jury indicted the defendant and the case went to trial. At trial, the jury was unable to reach a verdict. After this trial, the county prosecutor again tried the case before a jury; in this instance, the jury acquitted the defendant
of all charges. At a third trial, the county prose- cutor was finally successful in having the defen- dant convicted. The defendant appeals on double jeopardy grounds.

Which of the following is the strongest reason for overturning the conviction on appeal?

(A) The magistrate determined that there was insufficient evidence to prosecute.

(B) The first grand jury refused to indict the defendant.

(C) The first trial had ended in a hung jury. (D) The second trial had ended in an acquittal

A

(D) The strongest reason for overturning the conviction on appeal is that the second trial had ended in an acquittal. The Fifth Amendment right to be free of double jeopardy for the same offense has been incorporated into the Fourteenth Amendment. The general rule is that once jeopardy attaches, the defendant may not be retried for the same offense. In jury trials, jeopardy attacheswhen the jury is empanelled and sworn, but the state may retry a defendant even if jeopardy has attached when the first trial ends in a hung jury. Hence, (C) is incorrect. (A) is incorrect. Because the jury had not been empanelled and sworn when the magistrate had found insufficient grounds to prosecute, no jeopardy had attached yet. Likewise, the jury deciding the case would not have been empanelled or sworn when the case was presented to a grand jury. Thus, jeopardy had not attached when the case was presented to the grand jury. As a result, (B) is incorrect. (D) is correct because the jury would had to have been empanelled and sworn (and thus jeopardy had attached), and no exception exists for a case ending in an acquittal.

33
Q

A daughter owed her father $1,250. The father’s best friend was having financial difficul- ties and the father wanted to help him, so the father told his daughter to pay the $1,250 to his friend when the debt came due in three days. Immediately after directing his daughter to pay his friend, the father called his friend and told him he should expect to get $1,250 from his daughter in three days. When the debt came due, the daughter tendered the $1,250 to her father instead of to his friend, and the father accepted the money. The friend sues the daughter for $1,250.

Which of the following is the most likely result?

(A) The friend will recover, because the father effectively assigned his right to collect the $1,250 to his friend.

(B) The friend will not recover, because the father’s acceptance of $1,250 from his daughter revoked his gift to his friend.

(C) The friend will not recover, because the daughter was never indebted to him and cannot be forced to pay him.

(D) The friend will not recover, because the daughter’s tender of $1,250 to her father, and her father’s acceptance of the money, constituted a novation.

A

LOOK at THIS CLOSELY

(B) The father validly assigned his right to receive the money to his friend. However, this assignment was revocable, and it was revoked when the father accepted the money from his daughter. The father’s right to receive the money from his daughter was a right that could be assigned. By telling her to pay the money to his friend, the father manifested an intent to transfer his rights completely and immediately to his friend. Neither a writing nor consideration was required for this assign- ment to be valid. However, these factors do affect revocability. This assignment was not given

for value. Such a gratuitous assignment is generally revocable. Among the exceptions to this rule are situations where the obligor has already performed, or where the assignor is estopped from revoking by virtue of the fact that he should reasonably foresee that the assignee will change his position in reliance on the assignment and such detrimental reliance occurs. Here, the daughter (the obligor) has not already performed the terms of the assignment. On the contrary, she tendered performance directly to the original obligee. Also, there is no indication that the friend in fact changed his position detrimentally in reliance on the assignment. Consequently, the general

rule of revocability of a gratuitous assignment applies. One way in which a gratuitous revocable assignment may be terminated is by the assignor taking performance directly from the obligor. By accepting the money from his daughter, the father (the assignor) took direct performance from the obligor, thereby revoking the assignment. As a result, his friend has no right to the money. (A) incorrectly fails to account for the fact that, although the father effectively assigned his right, he later revoked this assignment

34
Q

Does a firm offer need to be in writing?

A

YUP

35
Q

[so this came up quite a bit]

A consumer properly sued a used car dealer- ship for fraudulent misrepresentation and breach of contract in federal court. The case proceeded through trial, and following closing arguments, the judge presented the jury instructions to the jury, but failed to give one of the jury instruc- tions requested by the consumer. The consumer’s attorney, however, did not object. Instead, he decided to wait and see how the jury would decide the case. After deliberating, the jury returned a verdict for the car dealership. The consumer filed a timely appeal based on the judge’s failure to give the requested jury instruc- tion, alleging that this failure caused the jury to render its verdict in favor of the car dealership.

How will the appellate court likely rule?

(A) Hear the consumer’s appeal, because the judge’s failure to give the requested in- struction amounted to plain error under the “plain error doctrine.”

(B) Deny the appeal, because the judge’s failure to give the requested instruction amounted to harmless error.

(C) Deny the appeal, because the consumer waived her right to appeal when she did not timely object to the judge’s failure to give the requested instruction at trial.

(D) Deny the appeal, because once the jury verdict was rendered, the jury instruction issue was moot.

A

(C) The court will likely deny the appeal. If a party does not give the trial judge a chance to consider and rule on an issue, the party generally cannot appeal the issue. Procedural mistakes (i.e., matters other than legal theories) are asserted by parties by raising a timely objection at trial. Failure to timely object waives the right to raise that issue on appeal. Here, failure to present the requested jury instructions is a procedural error, and the failure of the consumer’s attorney to object to the issue at trial waived the issue from being raised on appeal.

36
Q

A state’s pension program provided supple- mental state pension benefits to surviving spouses and children of state employees. The program provided that when the spouse remar- ried, that spouse’s benefits would be gradu-
ally terminated based on a statutory formula. Because of statistics showing past disparities between the household income levels of male surviving spouses and female surviving spouses, different formulas were used for the termination schedule depending on whether the surviving spouse was male or female.

A widower of a state employee was informed after he remarried that his pension benefits would be terminated in 90 days according to the applicable formula. Upon learning that a similarly situated widow would have continued to receive benefits for six months after remar- rying, the widower decided to file suit in federal court, alleging that the state program is uncon- stitutional because it is discriminatory and it unfairly burdens his right to marry.

Which of the following best states the burden of persuasion in this case?

(A) The state must demonstrate that the pro- gram is narrowly tailored to achieve a compelling government interest.

(B) The state must demonstrate that the program is substantially related to an important government interest.

(C) The widower must demonstrate that the program is not substantially related to an important government interest.

(D) Thewidowermustdemonstratethatthe program is not rationally related to a legiti- mate government interest

A

This is about equal protection not due process because the right is not directly affected.

While marriage is a fundamental right, strict scrutiny applies only to legislation that directly and substantially interferes with the right to marry. Laws terminating certain benefits upon marriage do not directly and significantly interfere with that right, and thus are not subject to strict scrutiny. so B

37
Q

Congress passed a statute designed to make college tuition-free. The statute significantly increased funding to states for this purpose; however, it made all federal funding to states dependent on the state making state universities tuition-free to United States citizens, and several state budgets consist of as much as 50% federal funding. Some states do not wish to participate in the program and want to retain their level of federal funding that existed before the statute was passed.

If these states challenge the constitutionality of the statute in court, will the court likely uphold the statute?

(A) Yes, because Congress can discriminate based on citizenship so long as it has a rational basis.

(B) Yes, because Congress can put conditions on funding, and the statute does not require discrimination based on citizenship.

(C) No, because the Equal Protection Clause prohibits discrimination based on citizen- ship.

(D) No, because the funding condition is unduly coercive.

A

(D) Thecourtwillnotupholdthestatute.Congressmayregulatestatesthroughthespendingpowerby imposing conditions on the grant of money to state governments. These conditions do not violate
the Tenth Amendment merely because Congress lacked the power to directly regulate the activity
so long as the conditions are clearly stated, relate to the purpose of the program, and are not unduly coercive. Here, the condition is unduly coercive because the funding amounts to as much as 50%
of states’ total budgets.

38
Q

A landowner had a contract to sell land to a developer. Before closing on the sale, the devel- oper died. In his will, he left his real property to his son and the residuary estate, including all of his personal property, to his daughter. Just after the developer died, a third party offered to purchase the land from the landowner at

a higher price than the contract price. The landowner notified the developer’s son that
he was canceling the contract, and would not be conveying the land to the developer’s son. The jurisdiction retains the common law rules regarding exoneration.

What are the rights of the developer’s son?

(A) He can demand conveyance of the property.

(B) He can demand conveyance of the property and compel the daughter to pay the purchase price.

(C) He can recover money damages from the landowner but is not entitled to demand conveyance of the property.

(D) He has no rights against the landowner, because the landowner’s personal property interest in receiving the sale proceeds entitled the landowner to find another purchaser upon the developer’s death.

A

(B) The developer’s son can demand conveyance of the property at closing and compel the daughter to pay the purchase price. Once the parties have entered into a valid land sale contract, the rights of the parties are fixed according to the doctrine of equitable conversion. The purchaser is regarded as the owner of the real property, and the seller is regarded as having a personal property right to the proceeds of the sale. The doctrine of equitable conversion governs the rights of the parties in a case where, as here, the purchaser dies before the sale is completed at closing. A deceased buyer’s interest passes as real property to the taker of the buyer’s real property interests, in this case the son. He can compel specific performance and demand a conveyance of the land. Furthermore, under the traditional common law rule, he is entitled to exoneration out of the personal property estate. Thus, he can compel the daughter, as taker of the developer’s personal property interests, to pay the purchase price out of her share of the estate.

39
Q

A plaintiff filed a negligence action against
a defendant in federal district court, seeking damages for personal injuries suffered in a traffic accident. The plaintiff timely served on the defendant a request for production of certain documents. The defendant objected to part of the request, claiming that it sought information beyond the scope of discovery. The defendant honestly believed this to be true, but case law clearly held to the contrary. After conferring with the defendant and not resolving the matter, the plaintiff filed a motion to compel the defen- dant to produce the documents.

If the court orders the defendant to produce the documents, what sanctions may the court impose on the defendant at that time?

(A) The court may enter a default judgment against the defendant.

(B) The court may strike relevant pleadings of the defendant or enter an order precluding the defendant from offering evidence on certain claims or defenses.

(C) The court may not impose sanctions at this time, but it must require the defendant to pay the reasonable expenses incurred by the plaintiff in making the motion to compel.

(D) The court at this time may neither impose sanctions nor require the defendant to pay any expenses incurred by the plaintiff.

A

(C) The court may not impose sanctions on the defendant, but it must require the defendant to pay the plaintiff’s reasonable expenses incurred in making the motion. If a motion to compel is granted, the court must require the opposing party to pay the movant’s reasonable expenses incurred in making the motion.

However, the court may not order this payment if the movant filed the motion before attempting to seek production without a court order, if the nondisclosure was substantially justified, or if other circumstances exist that make an award of expenses unjust. [Fed. R. Civ. P. 37(a)(5)(A)] None of these exceptions appear applicable here.

(A) and (B) are incorrect because they describe possible sanctions for a party who fails to comply with an order to provide discovery.

40
Q

The plaintiff sued the defendant dry cleaner, claiming that it had permanently ruined her $10,000 mink coat by cleaning it with a solvent that left an extremely offensive odor that smelled like “skunk.” Further attempts to have the odor removed by other cleaning services were unsuc- cessful. The odor was so bad that she could no longer wear the coat.

At the trial, the plaintiff testified to the above facts. She then identified a mink coat as her coat that the defendant had ruined. She testified that it still smelled the same as it did after the defendant had cleaned it. The plaintiff’s counsel offered to introduce the coat for the purpose

of having the jury smell it. Defense counsel objected.

How should the court rule?

(A) The coat is admissible based on the plain- tiff’s testimony.

(B) The coat is admissible, but the plaintiff must first present extrinsic evidence suffi- cient to support a finding that the coat is the coat that she had cleaned by the defendant.

(C) The coat is not admissible because the plaintiff’s testimony has not been impeached.

(D) The coat is not admissible because its limited probative value in resolving the case would be substantially outweighed by the prejudice that would result from the jury smelling the coat.

A

(A) The court should admit the coat based on the plaintiff’s testimony. Federal Rule of Evidence 402 provides that all relevant evidence is admissible unless a specific rule keeps the evidence out or limits its admissibility. In the case of real evidence, the object at issue is presented for inspection by the trier of fact. Such evidence can be presented to any of the senses of the jury from which the jury can obtain relevant information. Clearly the odor of the coat is a central issue in the case and the jury would obtain relevant evidence on that issue by smelling the coat. Admitting the coat for the stated purpose would violate no other rules of evidence. (B) is wrong because the plain- tiff’s testimony is sufficient authentication. Federal Rule 901 does require, as a prerequisite to the admission of real proof, evidence sufficient to support a finding that the item is what the propo- nent claims. However, real evidence is commonly authenticated by recognition testimony, such as in the case here. Thus, Rule 901 has been satisfied by the plaintiff’s testimony, and additional evidence would not be required.

41
Q

*

The National Park Service recently created a new personnel level for field employees, which became the highest salaried position available to Park Service field employees. The position is restricted to employees over six feet in height. A female ranger who is five feet, three inches tall seeks your advice as to whether she can challenge the validity of the height restriction in federal court.

If you decide to file suit on her behalf,
which of the following would be your strongest argument against the validity of the restriction?

(A) Because most women are less than six feet tall, the restriction is unconstitutional as a violation of the Equal Rights Amendment.

(B) Because most women are less than six feet tall, the restriction is an invalid discrimi- nation on the basis of gender in violation of the Due Process Clause of the Fifth

A mendment.

(C) Because most women are less than six feet tall, the restriction is an invalid genderbased discrimination in violation of the Equal Protection Clause of the Fourteenth Amendment.

(D) The restriction denies the ranger a property right without an opportunity for a hearing before a neutral decisionmaker, in violation of the Due Process Clause of the Fifth Amendment.

A

So it was not EP because EP applies only to states and NOT to the federal government.

C) is incorrect because the Fourteenth Amendment is applicable only to states and not to the federal government. Because an agency of the federal government is being sued, the ranger must rely on the Fifth Amendment.

(B) The ranger’s strongest argument, although by no means guaranteed of success, is that the height restriction is a gender-based classification that is not substantially related to important govern- mental interests. The Due Process Clause of the Fifth Amendment protects against action by the federal government. Although not expressly stated, this clause also provides an equal protection guarantee against federal action that generally applies to a similar extent that the Fourteenth Amendment Equal Protection Clause applies to the states. If the ranger can show that the Park Service restriction actually establishes a classification of eligibility for the new position based on gender, then the restriction will be found to violate the Fifth Amendment Due Process Clause unless the government has an exceedingly persuasive justification that the restriction is substan- tially related to important governmental interests. However, if the ranger is only able to show that the restriction has a discriminatory impact without being able to prove discriminatory intent, the court will not treat it as a gender-based classification and the ranger will not be successful. Despite the difficulty of success, however, (B) is the correct

42
Q

A builder entered into a contract with a landowner to build a warehouse for $500,000
by August 1. The agreement provided for five progress payments of $100,000 each at various stages of completion. On June 20, after the builder had spent $350,000 on performance
and received $300,000 in progress payments, the builder notified the landowner that he
was quitting the project. The landowner hired another contractor to complete the warehouse by August 1 for $250,000, which was a reasonable price given the short deadline.

Which of the following statements regarding the parties’ remedies is correct?

(A) The builder can recover $50,000, the differ- ence between the amount he expended on performance and the amount he was paid, to prevent the landowner’s unjust enrich- ment.

(B) Neither party can recover anything, because the $50,000 extra that the landowner had
to pay to complete the building is offset
by the $50,000 difference between the builder’s expenditures and the payments the landowner made to him.

(C) The landowner can recover $50,000, the difference between the contract price and the total amount he paid for completing the building.

(D) The landowner can recover $100,000, the difference between the contract price and the total amount spent constructing the building.

A

C) The landowner can recover $50,000, which is the amount above the contract price that it will cost to get the building completed. In construction contracts, the standard measure of damages when the builder breaches will depend on when the breach occurred. If the builder breaches after partially performing, the owner is entitled to the cost of completion plus reasonable compensation for any delay in performance (unless completion would involve undue economic waste). Here, the cost of completion (the amount above the contract price that it will cost to get the building completed) is $50,000, which was a reasonable price considering the deadline. Hence, that is what the landowner can recover. Most courts will allow the builder to offset or recover for work performed to date if necessary to avoid the unjust enrichment of the owner. (A) and (B) are incorrect because the landowner is not being unjustly enriched by the additional amount that the builder expended in performance over the progress payments that it received. Thus, the builder is not entitled to recover the $50,000 he spent above the amount he was paid. The landowner still had to pay $50,000 more than the contract amount for completion of the warehouse because of the builder’s breach; thus, that is the landowner’s recovery.

(On the other hand, if the cost of completing the building to specifications were only $150,000 after the builder’s breach, the builder could recover $50,000 in restitution from the landowner in a quasi-contract action because the landowner would have been unjustly enriched from the builder’s breach.)

43
Q

A parcel of property was devised to a husband and a wife “as joint tenants with right of survi- vorship” through the will of the husband’s mother. After title had passed to them, the husband and the wife experienced marital diffi- culties and legally separated. Unbeknownst to the husband, the wife quitclaimed her interest in the property to a bona fide purchaser for value. Shortly thereafter, the husband and the wife reconciled. The next month, the wife was killed in an auto accident. The purchaser of the wife’s interest filed a suit for partition of the property. The husband filed an appropriate counterclaim for quiet title, asserting that he was owner of the entire parcel by right of survivorship.

How should the court rule?

(A) For the purchaser, because he owns an un- divided one-half interest in the property.

(B) For the purchaser, because the husband and the wife are presumed to have taken title from the mother as tenants in common under modern law.

(C) For the purchaser, because the husband and the wife were legally separated when he purchased his interest from the wife.

(D) For the husband, because he succeeded to the entire ownership when the wife died.

A

SO, apparently most states do not recognize tenancy by the entirety and also the couple here did not take their tenancy by virtue of being married.

C) is a trap for the unwary, because it implies that the marital status of the husband and the wife is relevant to the ownership of this property. Under the common law, a grant to husband and wife resulted in the creation of a tenancy by the entirety, which carried with it a right of survivorship that would not be eliminated by a conveyance to an outsider by one spouse alone. Divorce can terminate a tenancy by the entirety (leaving the parties as tenants in common with no right of survivorship), but separation does not terminate the estate. Hence, if this were a tenancy by the entirety, the husband would prevail rather than the purchaser. However, most states do not recog- nize a tenancy by the entirety. As noted above, the husband and the wife took the property as joint tenants. The husband and the wife did not receive this status by virtue of the fact that they were married, nor is their continuation in such status dependent on their remaining married. The wife’s conveyance of her interest to the purchaser severed her joint tenancy with the husband and elimi- nated the right of survivorship, regardless of whether she and her husband were legally separated at the time.

A) The wife’s conveyance of her interest in the property to the purchaser severed the joint tenancy between the husband and the wife, leaving the husband and the purchaser holding the entire parcel as tenants in common. A joint tenancy is a type of concurrent ownership of a parcel of land that is distinguished primarily by the right of survivorship; i.e., when one joint tenant dies, the property is freed of her concurrent interest and the survivor retains an undivided right in
the property that is no longer subject to the interest of the deceased co-tenant. To create a joint tenancy, a grantor must explicitly indicate in the conveyance that the parties are to hold as joint tenants. If one joint tenant conveys her interest, the joint tenancy is severed. The new tenant holds as a tenant in common with the remaining joint tenant, so that there is no longer a right of survivorship. Here, the mother, grantor of the property, expressly indicated in her will that the husband and the wife were to take the parcel of property as joint tenants with right of survivorship. When the wife quitclaimed her interest in the property to the purchaser, the joint tenancy between the husband and the wife was severed (despite the fact that the husband did not know of the conveyance to the purchaser). This left the husband and the purchaser holding the property as tenants in common, with no right of survivorship. The husband and the purchaser each own an undivided one-half interest in the property, and the purchaser will succeed in his suit to partition the property to reflect the respective interests of the co-tenants.

44
Q

A defendant was charged with robbery and felony murder based on a death that arose during the robbery. The defendant pleaded not guilty and insisted on a jury trial. Right before the
trial began, he fired his attorney and decided
to defend himself. The court made a finding
that the defendant was competent to represent himself at trial. The defendant then insisted
on trying both of his charges separately in two different trials. The trial judge asked the defen- dant if he was confident that he wanted to have two separate trials. The defendant replied: “I am, your Honor.” The felony murder case was tried first, and the jury found the defendant not guilty. The defendant then moved to dismiss the robbery charge based on double jeopardy.

How should the court rule on the defendant’s motion?

(A) Grant the motion, because double jeopardy forbids a second trial relitigating a lesser included offense.

(B) Grant the motion, because the defendant’s Sixth Amendment right to counsel was violated.

(C) Deny the motion, because robbery is not a lesser included offense of felony murder.

(D) Deny the motion, because the charges could have been tried together but the defendant consented to the separate trials.

A

(D) The court should deny the motion. Where charges can be tried in a single trial, but the defendant consents to having two separate trials, there is no double jeopardy violation. Just as consent is an exception to the warrant requirement, consent to a second trial is an exception to the double jeopardy rule.

45
Q

A motorist from State A struck and injured a pedestrian in State B. The pedestrian, a State B resident, brought an action in a State B federal court against the State A motorist, seeking $100,000 in damages. The summons and complaint were served on a receptionist at the motorist’s place of business in State A. State A’s rules permit service of process in this manner, while State B’s rules do not.

If the motorist moves to dismiss the complaint on the basis of improper service of process, is the court likely to dismiss the action?

(A) Yes, because, under choice of law rules, the court will apply the law that a state court in State B would apply.

(B) Yes, because the federal rules do not permit service on an individual defendant by deliv- ering process to a third party found at the defendant’s place of employment.

(C) No, because the federal rules permit service under the rules of the state in which service will be effected.

(D) No, because the federal rules permit service on a person of suitable age and discretion at the defendant’s place of employment.

A

C) The court is not likely to dismiss the action. Generally, Rule 4 allows for: (i) personal service, (ii) service left at the defendant’s usual place of abode with one of suitable age and discretion residing therein, or (iii) service upon an authorized agent of the defendant. Alternatively, service may be made as provided by the rules of the state in which the federal court sits or the state in which service is to be effected, regardless of the basis of subject matter jurisdiction. Here, the rules of State A, the state in which service was effected, permit service of process in this manner. Hence, service of process was proper.

46
Q

A liquor store owner sued a woman for negli- gence in federal court after she drove her car through the front of his store. At trial, the store owner did not present any evidence on the issue of causation. No motions were filed during the trial, and the jury returned a general verdict for the plaintiff. Immediately after the verdict was read, the defendant filed a motion for judgment as a matter of law and a motion for a new trial.

The court may:

(A) Deny the motion for a new trial, but grant the motion for judgment as a matter of law.

(B) Deny both the motion for a new trial and the motion for judgment as a matter of law.

(C) Grant either the motion for judgment as a matter of law or the motion for a new trial, but not both.

(D) Grant the motion for a new trial, but not the motion for judgment as a matter of law.

A

Against “great weight of the evidence”

(D) A motion for a new trial may be granted because of an error during the trial (usually going to the admissibility of evidence or the propriety of the jury instructions), because the verdict is against the weight of the evidence (limited to cases where the judge finds the verdict seriously erroneous), because of jury misconduct, or because the verdict is excessive or inadequate. A motion for judgment as a matter of law (formerly known as a motion for directed verdict) may be made by any party any time before submission of the case to the jury. Here, the court may grant the motion for a new trial under these circumstances because there was absolutely no evidence presented on the issue of causation, which is a fundamental element of a negligence claim that must be proven. As such, the court may find that the verdict is seriously erroneous and against the weight of the evidence and grant the motion for a new trial. However, the court cannot grant the motion for judgment as a matter of law because it was filed after the case was submitted to the jury, so it was too late for such a motion.

47
Q

A seller owned a large parcel of land. The western half was undeveloped, and the eastern half contained a grove of apple trees. The seller gave a buyer a deed conveying “the western half of the parcel from the western boundary to the grove of apple trees, comprising 220 acres.” It was subsequently determined by survey that the land conveyed to the buyer was in fact 229 acres.

In a dispute between the seller and the buyer as to the mistake, which of the following is most accurate?

(A) The deed is invalid because of the mutual mistake of the parties.

(B) The deed is invalid unless the court admits parol evidence as to the amount of acreage conveyed.

(C) The deed is valid, and the buyer is the owner of 220 acres.

(D) The deed is valid, and the buyer is the owner of 229 acres.

A

so one of the rules of construction apparently is that a physical description take precedence over quantity.

(D) The deed is valid and the buyer owns 229 acres. When there is a mistake or inconsistency in the description of property in the deed, one of the rules of construction is that the physical descrip- tion takes precedence over the quantity description unless there are grounds for reformation of the deed.

Reformation is an equitable action in which the court rewrites the deed to make it conform to the intention of the parties. It is granted when the deed does not express the agreement of the parties due to mutual mistake or a scrivener’s error, and may also be granted when there is a unilateral mistake if misrepresentation is involved. Here, the facts indicate that the seller and the buyer were bargaining for a specific physical location (“the western half of the parcel from the western boundary to the grove of apple trees”) and not for a specific number of acres. Thus, there appear to be no grounds for reformation. A conflict in description does not invalidate a deed, so (A) and (B) are incorrect. While parol evidence may be admissible to ascertain the parties’ intent, the absence of parol evidence will not invalidate the deed as long as rules of construction may

be applied to resolve any inconsistency. (C) is incorrect because, as discussed above, physical descriptions prevail over quantity.

48
Q

A woman chastised her roommate when she saw that neither the roommate nor her boyfriend wore a helmet when they rode on the boyfriend’s motorcycle. The roommate said that helmets were too restricting. The woman’s brother had died in a motorcycle accident because he had not worn a helmet, so she decided to do something to make a lasting impression on her roommate. She called her roommate at work one day and left a message that the roommate’s boyfriend was in a motorcycle accident and was in the hospital on life support. The roommate was very upset when she got the message and left immedi- ately for the hospital. When she found out later that the message was not true, she became even more upset.

If the roommate brings an action against the woman to recover for her emotional distress, is she likely to prevail?

(A) Yes, because it was foreseeable that the roommate would suffer severe emotional distress.

(B) Yes, because the woman knew that there was a high likelihood that the roommate would suffer severe emotional distress.

(C) No, because it does not appear that the roommate suffered physical injury from her distress.

(D) No, because the roommate and her boyfriend were not related.

A

So here it was actually not a bystander case because the woman intended to cause emotional harm to the person

(B) The roommate is likely to prevail because the woman acted in reckless disregard of a high probability that emotional distress would result. To establish a prima facie case for intentional infliction of emotional distress, plaintiff must show: (i) an act by defendant amounting to extreme and outrageous conduct, (ii) intent on the part of defendant to cause plaintiff to suffer severe emotional distress, or recklessness as to the effect of defendant’s conduct, (iii) causation, and (iv) damages—severe emotional distress. Here, the woman’s conduct was extreme and outrageous, and she caused the roommate to suffer severe emotional distress. The facts indicate that she wanted to make a lasting impression on her roommate, so she knew that there was a high likelihood that the roommate would suffer severe distress; hence, she has acted with the mental state of recklessness, and the roommate can establish the prima facie case for this tort.

49
Q

The legislature of a state was concerned that the numerous and strident television, radio,
and newspaper advertisements by auto dealer- ships annoy and mislead the public. Therefore,
it enacted comprehensive legislation regulating the timing and content of such ads, limiting their duration, frequency, and the types of claims and information made and given.

Which of the following statements is most accurate as to the constitutionality of the state’s ad regulation?

(A) It is unconstitutional, because it infringes on the First and Fourteenth Amendment rights of auto dealers to free speech.

(B) It is constitutional if it does not prohibit the dissemination of truthful information about price and the availability of products, and is narrowly tailored to serve a substantial government interest.

(C) It is constitutional, because it is within the police power of the state and no federal constitutional rights are infringed.

(D) It is unconstitutional, because it infringes on the rights of the auto dealers to enter into contracts for advertising.

A

B) Of all the alternatives, statement (B) most accurately reflects the balance between the scope
of First Amendment protection for the dissemination of truthful commercial speech and the state’s ability to enact narrowly drawn regulations to advance substantial governmental interests. Although commercial speech is protected by the First Amendment, it is subject to significant regulation. A state may outlaw commercial speech that proposes an unlawful transaction or that is misleading or fraudulent. If commercial speech concerns a lawful activity and is not misleading or fraudulent, the government regulation, to be valid, must directly advance a substantial govern- mental interest and must be no more extensive than necessary to serve that interest. The regula- tion must be narrowly drawn and there must be a reasonable fit between the legislation’s end and the means chosen. If, as (B) states, the legislation here at issue does not prohibit the dissemination of truthful information about prices and product availability, and is otherwise narrowly tailored

to serve a substantial state interest, the legislation probably will constitute a valid regulation of commercial speech. (A) is incorrect because it would overly limit the ability of the state to reason- ably regulate commercial speech. This legislation does not necessarily violate auto dealers’ rights of free speech. If it does not prohibit the dissemination of truthful information and is otherwise reasonable, the legislation is sufficiently narrow to pass constitutional muster.

50
Q

A chef wanted to open his own restaurant
and a contractor offered to build the place for $160,000. Their written contract provided that the chef would pay the contractor $60,000 in cash when construction commenced, scheduled for April 15 after the spring thaw. On completion of the restaurant on September 30, the contractor would be paid the remaining $100,000. The region had a late spring, and on April 30 the contractor had not yet commenced construction of the restaurant.

Has the contractor breached the contract?

(A) No, and the chef need not make the initial $60,000 payment.

(B) No, but the chef must make the initial $60,000 payment.

(C) Yes, in a nonmaterial particular; thus, the chef need not make the initial $60,000 payment.

(D) Yes, in a material particular; thus, the chef may treat the contract as at an end and sue for damages.

A

(A) The contractor has not breached the contract, and the chef need not make the first payment until the contractor begins work. The contractor promised to build the restaurant by September 30. He did not promise to begin on April 15. The contractual term as to the contractor’s begin- ning construction is a condition precedent to making the first payment. It is a condition relating to the chef’s performance, not the contractor’s performance. Nothing in the facts indicates that the contractor promised to begin on April 15. Contracts are construed as a whole and words are

given their ordinary meaning. The purpose of the contract is to build a restaurant by September 30. Construction cannot commence before the spring thaw. Thus, the best interpretation is that
the language regarding commencement of construction was merely a condition of the chef’s first payment, inserted to insure that the contractor was motivated to begin and that the chef would
not be out of pocket if the contractor failed to begin. Thus, the term regarding the beginning of construction of the restaurant on April 15 merely fixes a tentative time of the start of performance, and does not involve an absolute promise by the contractor to commence performance on April 15. Because the contractor was under no absolute duty to commence construction on April 15,

his failure to do so does not constitute a breach of the contract.

51
Q

A customer slipped and fell in a store, suffering a severe injury. Several weeks after the accident, anticipating that the customer would file an action against it, the store’s attorney had the store manager interview any employees who were near the accident to determine what they saw or heard. The store manager did so, taking handwritten notes. The notes are now in the store’s possession. The customer subsequently filed a civil action against the store in federal district court. The complaint alleged that the store negligently left a spill on the floor of the store, causing the customer’s fall. The customer’s attorney served on the store a request for produc- tion of documents, which included a request for all documents and reports prepared by the store that relate to the customer’s fall and injury.

Must the store produce to the customer the notes taken by the store manager when he inter- viewed the store’s employees?

(A) Yes, because the request seeks information that is relevant to the claim or defense of a party, and the manager’s notes fall squarely within the scope of the request.

(B) Yes, because the manager’s notes were not taken by the store’s attorney and thus are not protected by the work product doctrine.

(C) No, because the manager’s notes are protected by a qualified immunity from discovery under the work product doctrine.

(D) No, because the manager’s notes are completely immune from discovery under the work product doctrine.

A

(C) The store does not have to produce the store manager’s notes because they are protected by a qualified immunity from discovery under the work product doctrine. Documents prepared in anticipation of litigation by a party or its representative are not discoverable unless the opposing party can show substantial need and that it cannot obtain the materials in an alternative way without undue hardship. (A) is incorrect because the manager’s notes are protected work product despite being relevant to the claim or defense of a party.

52
Q
A